Difference between revisions of "2007 IMO Problems/Problem 1"
m (→Solution) |
Not detriti (talk | contribs) m (added a comment to solution 2) |
||
(One intermediate revision by the same user not shown) | |||
Line 19: | Line 19: | ||
<hr> | <hr> | ||
− | ==Solution== | + | ==Solution 1== |
− | Since <math>d_i=\max\{a_j:1\le j\le i\}-\min\{a_j:i\le j\le n\}</math>, all <math>d_i</math> can be expressed as <math>a_p-a_q</math> | + | Since <math>d_i=\max\{a_j:1\le j\le i\}-\min\{a_j:i\le j\le n\}</math>, all <math>d_i</math> can be expressed as <math>a_p-a_q</math> where <math>1\le p\le i\le q \le n</math>. Thus <math>d</math> can be expressed as <math>a_p-a_q</math> for some <math>p</math> and <math>q</math> with <math>1\le p\le q\le n</math> |
− | + | <b>Lemma: </b> <math>d\ge 0</math>. | |
− | |||
− | |||
− | |||
− | |||
− | <b>Lemma | ||
− | |||
− | |||
− | |||
− | |||
− | |||
− | |||
− | |||
− | |||
− | < | + | Since <math>a_p \geq a_i</math> and <math>a_i \geq a_q</math> for some <math>i</math> satisfying <math>p \leq i \leq q</math> it follows <math>d \geq 0</math>. |
− | <b>a) </b> | + | <b>(a): </b> |
− | <b>Case </b> | + | <b>Case </b> <math>d=0:</math> |
If <math>d=0</math>, <math>\max\{|x_i-a_i|:1\le i\le n\}</math> is the maximum of a set of non-negative number, which must be at least <math>0</math>. | If <math>d=0</math>, <math>\max\{|x_i-a_i|:1\le i\le n\}</math> is the maximum of a set of non-negative number, which must be at least <math>0</math>. | ||
− | + | <b>Case </b> <math>d>0:</math> | |
− | |||
− | <b>Case </b> | ||
− | |||
− | |||
− | |||
− | |||
− | |||
− | |||
− | |||
− | |||
− | + | Assume for the sake of contradiction that <math>\max\{|x_i-a_i|:1\le i\le n\}<\dfrac{d}{2}</math>. | |
− | + | Then <math>\forall i</math>, <math>|x_i-a_i|<\dfrac{d}{2}</math>. So <math>|x_p-a_p|<\dfrac{d}{2}</math> and <math>|x_q-a_q|<\dfrac{d}{2}</math>. | |
− | <cmath>x_p-x_q>a_p-a_q-d=a_p-a_q-a_p+a_q=0</cmath> | + | Thus <math>x_p>a_p-\dfrac{d}{2}</math> and <math>x_q<a_q+\dfrac{d}{2}</math>. Subtracting the two inequalities we obtain |
− | + | <cmath> | |
− | <math>x_p>x_q</math> | + | x_p-x_q>a_p-a_q-d=a_p-a_q-a_p+a_q=0. |
+ | </cmath> | ||
+ | i.e. <math>x_p>x_q</math> which contradicts <math>x_p\le x_q</math> (since <math>p \leq q</math>). | ||
<br/> | <br/> | ||
− | Thus | + | Thus <math>\max\{|x_i-a_i|:1\le i\le n\}\ge\dfrac{d}{2}</math>. |
− | |||
− | |||
− | |||
− | <b>(b) </b> | + | <b>(b): </b> |
− | A set of <math> | + | A set of <math>x_i</math> where equality holds in (*) is: |
− | <cmath>x_i=\max\{a_j:1\le j\le i\}-\frac{d}{2}</cmath> | + | <cmath> |
− | + | x_i=\max\{a_j:1\le j\le i\}-\frac{d}{2} | |
− | Since <math>\max\{a_j:1\le j\le i\}</math> is a non-decreasing function, <math>x_i</math> is non-decreasing. | + | </cmath> |
+ | for all <math>i</math>. Since <math>\max\{a_j:1\le j\le i\}</math> is a non-decreasing function, <math>x_i</math> is non-decreasing. | ||
<br/> | <br/> | ||
− | <math>\forall | + | <math>\forall i</math> let <math>a_{m_i}=\max\{a_j:1\le j\le i\}</math>. Then <math>a_{m_i}-a_i<a_{m_i}-\min\{a_j:i\le j\le n\}=d_i</math>. |
− | |||
− | |||
− | Thus | + | Thus <math>0\le a_{m_i}-a_i \le d</math>. <math>(0\le a_{m_i}-a_i</math> because <math>a_m</math> is the max of a set including <math>a_i</math>). |
− | < | + | Therefore one has |
+ | <cmath> | ||
+ | -\dfrac{d}{2} \le a_{m_i}-a_i - \dfrac{d}{2} \le \dfrac{d}{2} | ||
+ | </cmath> | ||
+ | hence | ||
+ | <math>|x_i-a_i| = \left|a_{m_i}- \frac{d}{2} -a_i \right| \le \frac{d}{2}</math>. Lastly since <math>\max\{|x_i-a_i|:1\le i\le n\}\ge\dfrac{d}{2}</math> and <math>|x_i-a_i|\le \frac{d}{2} \ \forall i</math>, it follows <math>\max\{|x_i-a_i|:1\le i\le n\}=\dfrac{d}{2}</math>. | ||
− | |||
− | < | + | <hr/> |
− | + | This is written by Mo Lam--- who is a horrible proof writer, so please fix the proof for me. Thank you. O, also the formatting. | |
− | + | (edited by not_detriti) | |
− | < | + | <hr/> |
− | + | ==Solution 2== | |
+ | '''(a):''' Let <math>d_j</math> satisfy <math>d_j = d</math>. Then <math>\exists p,q</math> with <math>p \leq j \leq q</math> such that <math>d_j = a_p - a_q</math> (note <math>a_p \geq a_j \geq a_q</math>. Note this reasoning also implies <math>d_i \geq 0</math> for all <math>i</math>). And for any <math>x_p \leq x_q</math> if <math>x_p > a_p - d/2 = a_q + d/2</math> then <math>|x_q-a_q| > d/2</math> in which case we're done. So we may assume <math>x_p \leq a_p-d/2</math>. But then <math>|x_p-a_p| \geq d/2</math> and we're done. | ||
− | < | + | '''(b):''' Let <math>f(k) = \max\{a_i : 1 \leq i \leq k\} - \max\{a_i: 1 \leq i \leq k-1\}</math> for <math>k=2,...,n</math>. Then let <math>\{a_{i_1},...,a_{i_m}\} = f^{-1}(0,\infty)</math> such that <math>i_1 < \cdots < i_m</math>. Let <math>i_0 = 1</math>. Then let <math>j_l</math> be such that <math>i_l \leq j_l</math> and <math>d_{i_l} = a_{i_l} - a_{j_l}</math> for <math>l=0,...,m</math>. Note that <math>a_{i_0} < a_{i_1} < \cdots < a_{i_m}</math> and <math>a_{j_0} \leq a_{j_1} \leq \cdots \leq a_{j_m}</math> since <math>a_{j_l} = \min\{a_i: i_l \leq i \leq n\}</math>. |
− | + | Therefore if we set | |
+ | <cmath> | ||
+ | x_i = \frac{1}{2}\big(a_{i_l}+a_{j_l} \big)\ \text{if } i_l \leq i < i_{l+1} | ||
+ | </cmath> | ||
+ | and | ||
+ | <cmath> | ||
+ | x_i = \frac{1}{2}\big(a_{i_m} + a_{j_m}\big)\ \text{if } i_m \leq i | ||
+ | </cmath> | ||
+ | one will have <math>x_1 \leq x_2 \leq \cdots \leq x_n</math>. Furthermore if <math>i_l \leq i < i_{l+1}</math> then | ||
+ | <math>a_{i_l} \geq a_i \geq a_{j_l}</math> so that | ||
+ | <cmath> | ||
+ | a_{i_l}-x_i = \frac{1}{2}\big(a_{i_l}-a_{j_l}\big) \geq a_i - x_i \geq a_{j_l}-x_i = \frac{1}{2}\big(-a_{i_l}+a_{j_l} \big) | ||
+ | </cmath> | ||
+ | hence | ||
+ | <cmath> | ||
+ | |a_i-x_i| \leq \frac{1}{2}\Big(a_{i_l}-a_{j_l}\Big) = \frac{d_{i_l}}{2} \leq \frac{d}{2}. | ||
+ | </cmath> | ||
+ | by definition of <math>d</math>. A similar argument shows the above equation also holds if <math>i \geq i_l</math>. Since <math>i</math> was arbitrary, combined with (a) it follows equality holds in (*) for this choice of <math>x_1 \leq \cdots \leq x_n</math>. | ||
− | + | ~not_detriti | |
− | |||
{{alternate solutions}} | {{alternate solutions}} | ||
Latest revision as of 00:12, 27 July 2025
Contents
Problem
Real numbers are given. For each
(
) define
and let
.
(a) Prove that, for any real numbers ,
(b) Show that there are real numbers such that equality holds in (*)
Solution 1
Since , all
can be expressed as
where
. Thus
can be expressed as
for some
and
with
Lemma: .
Since and
for some
satisfying
it follows
.
(a):
Case
If ,
is the maximum of a set of non-negative number, which must be at least
.
Case
Assume for the sake of contradiction that .
Then ,
. So
and
.
Thus and
. Subtracting the two inequalities we obtain
i.e.
which contradicts
(since
).
Thus .
(b):
A set of where equality holds in (*) is:
for all
. Since
is a non-decreasing function,
is non-decreasing.
let
. Then
.
Thus .
because
is the max of a set including
).
Therefore one has
hence
. Lastly since
and
, it follows
.
This is written by Mo Lam--- who is a horrible proof writer, so please fix the proof for me. Thank you. O, also the formatting.
(edited by not_detriti)
Solution 2
(a): Let satisfy
. Then
with
such that
(note
. Note this reasoning also implies
for all
). And for any
if
then
in which case we're done. So we may assume
. But then
and we're done.
(b): Let for
. Then let
such that
. Let
. Then let
be such that
and
for
. Note that
and
since
.
Therefore if we set
and
one will have
. Furthermore if
then
so that
hence
by definition of
. A similar argument shows the above equation also holds if
. Since
was arbitrary, combined with (a) it follows equality holds in (*) for this choice of
.
~not_detriti
Alternate solutions are always welcome. If you have a different, elegant solution to this problem, please add it to this page.
Resources
2007 IMO (Problems) • Resources | ||
Preceded by First question |
1 • 2 • 3 • 4 • 5 • 6 | Followed by Problem 2 |
All IMO Problems and Solutions |
- <url>viewtopic.php?p=894656#p894656 Discussion on AoPS/MathLinks</url>